GMAT 考满分题库

GWD - 逻辑CR - 196
In Gandania, where the government has a monopoly on tobacco sales, the incidence of smoking-related health problems has risen steadily for the last twenty years. The health secretary recently proposed a series of laws aimed at curtailing tobacco use in Gandania. Profits from tobacco sales, however, account for ten percent of Gandania's annual revenues. Therefore, Gandania cannot afford to institute the proposed laws.

Which of the following, if true, most seriously weakens the argument?
  • AAll health care in Gandania is government-funded. 分析该选项
  • BImplementing the proposed laws is not likely to cause a significant increase in the amount of tobacco Gandania exports. 分析该选项
  • CThe percentage of revenue Gandania receives from tobacco sales has remained steady in recent years. 分析该选项
  • DProfits from tobacco sales far surpass any other single source of revenue for the Gandanian government. 分析该选项
  • ENo government official in Gandania has ever previously proposed laws aimed at curtailing tobacco use. 分析该选项
显示答案
正确答案: A

讨论题目 或 发起提问

|

题目讨论

  • 按热度
  • 按顺序

最新提问